Prep19; section2; 16 Forum

Prepare for the LSAT or discuss it with others in this forum.
Post Reply
soyeonjeon

New
Posts: 86
Joined: Tue Jul 31, 2012 12:28 am

Prep19; section2; 16

Post by soyeonjeon » Mon Oct 22, 2012 5:32 am

First, Can someone explain why A is not the answer? I think that although the police officer attacks the motorcycle driver's premise, I think that by doing so, the police officer implies that he disagrees with the driver's conclusion or sth short of agreeing at least.

Second, why B is not the answer?
How do B and C fundamentally differ?

Thanks.

Also, do high scorerea read all the answer choices mostly?
I only read A for this one and did not bother to look further because of time management and got it wrong. I do not have the time to read all the answers for some questions I think... CAN Someone elaborate on this problem as well??

I would be more than grateful Just to get answers to the problems

ws81086n

Bronze
Posts: 313
Joined: Sun Aug 26, 2012 9:47 pm

Re: Prep19; section2; 16

Post by ws81086n » Mon Oct 22, 2012 6:11 am

Nice question--speaks to the efficacy of old LR's for prep. I got the answer, though I am not sure I fully understand the logic of the question. But here's a try: (A) doesn't work because the police officer only discusses one negative effect of reducing the speed limit. There could be other effects of it that would be undesirable. (B) doesn't work because the MC Driver does not say in what manner the professional drivers will break the law, should the speed limit be reduced. Could be speed limits, could be something else. (C), since it is more general, using "break the law" instead of violate the speed limit, can encompass both statements. MC Driver clearly thinks it is the case, and the PO appears to not think so, since he thinks the speed limit will not be violated by anyone.

vegso

New
Posts: 67
Joined: Wed Aug 22, 2012 2:08 pm

Re: Prep19; section2; 16

Post by vegso » Mon Oct 22, 2012 7:43 am

yes read all the ACs so you dont fall for the easy A answer looks right trap

User avatar
boblawlob

Silver
Posts: 519
Joined: Mon Oct 11, 2010 7:29 pm

Re: Prep19; section2; 16

Post by boblawlob » Mon Oct 22, 2012 8:09 pm

To Paraphrase

Driver: Pro drivers drive all the time and are therefore competent. Therefore, don't lower speed limit because it will cause some people (i.e. pro drivers) to break the law. [Assumes pro drivers only drive fast and not any slower]

Bold: Conclusion

Cop: [I disagree with your premise.] You can control your speed as a driver and so saying a lower speed limit will cause people to break the law is incorrect.


A. Cop doesn't explicitly challenge conclusion, but challenges the premise. So he could still agree with conclusion, but just disagree with how the driver reaches it. Wrong
B. Cop doesn't say a driver WILL abide with speed limit. Only says that they CAN. So cop could still agree with driver in stating that pro drivers will violate speed limits. Wrong
C. Driver basically states this in his premise that follows the conclusion. Cop basically says the opposite in the 2nd part of his sentence.
D. Driver agrees with this statement and there's no indication the cop would think otherwise. Wrong
E. Drivers WISHING to follow speed limit is not discussed by either side. Wrong.

Want to continue reading?

Register now to search topics and post comments!

Absolutely FREE!


Post Reply

Return to “LSAT Prep and Discussion Forum”